If an activity significantly reduces chronic lower back pain, doctors should be prepared to discuss the merits of tha...

jack515 on December 24, 2018

sep. 2017

I wanted to know how B could be the answer. In the passage, it states that both activities lead to an equal reductions in back pain. Why then is it an assumption?

Replies
Create a free account to read and take part in forum discussions.

Already have an account? log in

jack515 on December 24, 2018

oh its significantly.

Katherine on December 24, 2018

Hi @jack515,

Based on your reply, it seems like you may have already figured out this question. Here is a brief explanation, just in case.

The passage says that doctors should be prepared to discuss the merits of an activity if it significantly reduces chronic lower back pain. Then it says that a study found that yoga and stretching with a physical therapist had equal reduction in chronic lower back pain. Note that it doesn’t state how much these activities were found to reduce pain (just a little or significantly?). Then the passage concludes that doctors must be prepared to discuss the merits of yoga with their patients.

In order for this conclusion to be true, we must assume that stretching with a physical therapist significantly reduces chronic back pain. Then, we would know that yoga also significantly reduces chronic back pain, because the study found that these activities had an equal reduction of pain. Therefore, this assumption is necessary in order for the conclusion of the argument to be true.